next up previous
Next: Question 3 Up: Solution to Assignment 2 Previous: Question 1

Question 2

Find the z-transforms of

(a) x=[1 0 0 1]

(b) x=[1 1 1 1 ...]

(c) x=[1 2 3 4 ...]

(d) x=[1 1/2 1/3 1/4 ...]

Solution.

(a) For a sequence of numbers x[n], the z-transform is written as:


\begin{eqnarray*}X \left (z \right) & = & \sum_{ k= - \infty }^{ \infty } x[k]z^...
...x[2]z^{-2}+x[3]z^{-3}\\
& = & 1+0+0+z^{-3}\\
& = & 1+z^{-3}
\end{eqnarray*}


(b) The signal x can be written as:

\begin{displaymath}x[n] = \left \{ \begin{array}{ll}
1 & \mbox{for $n \ge 0$ }\\
0 & \mbox{otherwise}
\end {array}
\right. \end{displaymath}

Thus

\begin{eqnarray*}X \left (z \right) & = & \sum_{ k= - \infty }^{ \infty } x[k]z^...
...^{-3}+....\\
& = & \frac{1}{1-z^{-1}}\\
& = & \frac{z}{z-1}
\end{eqnarray*}


for |z|>1.

(c) When we define unit step function

\begin{displaymath}u[n] = \left \{ \begin{array}{ll}
1 & \mbox{for $n \ge 0$ }\\
0 & \mbox{otherwise}
\end {array}
\right. \end{displaymath}

Then the signal x is equivalent to:

\begin{eqnarray*}x \left [n \right] & = & u[n] + u[n-1] + u[n-2] + ... \\
& = & \sum_{ k= - \infty }^{ \infty } u[n-k]
\end{eqnarray*}


Thus we can write

\begin{eqnarray*}X \left (z \right) & = & {\em z-transform}[\sum_{ k= 0 }^{ \inf...
...-k]]\\
& = & \sum_{ k= 0 }^{ \infty }{\em z-transform}[u[n-k]]
\end{eqnarray*}


From the time-shifting property of z-transform,

\begin{eqnarray*}X \left (z \right) & = & \sum_{ k= 0 }^{ \infty }z^{ -k}{\em z-...
...} \frac{1}{1-z^{-1}}\\
& = & \frac{1}{\left(1-z^{-1}\right)^2}
\end{eqnarray*}


for |z|>1.

(d) The signal x is equivalent to:

\begin{eqnarray*}x \left [n \right] & = & \frac{1}{n+1}u[n]\\
\end{eqnarray*}


From the definition of the z-transform,

\begin{eqnarray*}X \left (z \right) & = & \sum_{ k= -\infty }^{ \infty } \frac{1...
...-1}} (z^{-1} + \frac{z^{-2}}{2} + \frac{z^{-3}}{3} + \cdots) \\
\end{eqnarray*}


This expression can be simplified by using the series


\begin{eqnarray*}\sum_{ k= 1 }^{ \infty }\frac{y^k}{k} & = & \ln \frac{1}{1-y}
\end{eqnarray*}


Then X(z) can be expressed as

\begin{eqnarray*}X \left (z \right) & = & \frac{1}{z^{-1}} (\ln \frac{1}{1-z^{-1}})\\
& = & z \ln \frac{z}{z-1}
\end{eqnarray*}




Hyun Ho Jeon
2001-01-28